LSAT and Law School Admissions Forum

Get expert LSAT preparation and law school admissions advice from PowerScore Test Preparation.

 Administrator
PowerScore Staff
  • PowerScore Staff
  • Posts: 8916
  • Joined: Feb 02, 2011
|
#23776
Complete Question Explanation

Flaw in the Reasoning. The correct answer choice is (A)

Just because none of the foundation’s money would be used for the weapons-related research does not mean that the money would not indirectly assist the university in pursuing such research. What if the funding in question is directed towards a humanitarian project that is already fully funded by grants that have no-strings-attached, which frees up money be diverted towards weapons-related research instead? The university would still be true to their promise, and yet the foundation’s money would have indirectly assisted in the pursuit of weapons-related research. Answer choice (A) is therefore correct.

Answer choice (A): This is the correct answer choice. See discussion above.

Answer choice (B): The fact that other research projects pursued by the university might be weapons-related as well does not affect the conclusion of the argument. This answer choice is incorrect.

Answer choice (C): It is clear that the university made the promise with the specific intention of retaining the grant. The university’s motivations are irrelevant to understanding whether the foundation’s reasoning is valid.

Answer choice (D): Since the money will be used for a humanitarian purpose, there is no reason to suspect it will not be used at all. This answer choice alters the facts in the stimulus and is incorrect.

Answer choice (E): It is clear that the new objective of the grant is to benefit humanitarian research. The distinction between means and ends is irrelevant to the author’s conclusion. This answer choice is incorrect.
 desmail
  • Posts: 50
  • Joined: Jul 05, 2011
|
#1524
Hi,

I'm having some trouble understanding what answer E means. I kept it as a contender and don't really know why its the wrong answer.

Also, choice C is incorrect because the question is asking about the conclusion and choice C deals with the premises, right?

Thanks in advance!
Dana
User avatar
 Dave Killoran
PowerScore Staff
  • PowerScore Staff
  • Posts: 5853
  • Joined: Mar 25, 2011
|
#1528
Hi Dana,

The question stem asks about the reasoning underlying the conclusion, so it will address elements of the premises. Answer choice (C) strikes me as an answer that describes a flaw that didn't occur--the university made a specific promise about the use of money, and that was done in response to a threat the cancel the grant. Thus, it seems like instead overlooking the possibility that the university made that promise to avert the threat, the foundation understood that they did it on purpose.

Answer choice (E) basically says that if the means are humanitarian, then the end is humanitarian. I would say that is not an assumption of the argument, because the foundation now knows that some of the research is not for humanitarian purposes.

Does that help?
 lawschoolforme
  • Posts: 33
  • Joined: Oct 15, 2013
|
#12031
Hi,

I understand why A is right, but I don't understand why D is wrong.

What exactly is D saying anyways?

-lawschoolforme
User avatar
 Dave Killoran
PowerScore Staff
  • PowerScore Staff
  • Posts: 5853
  • Joined: Mar 25, 2011
|
#12038
Hi Lawschool,

Good question. Answer choice (D) is saying that the foundation makes the mistake of thinking that because the university said it wouldn't use the money on weapons research that it then won't use it at all. A similar mistake would occur if you had a friend who said, "I'm not spending my birthday money on a new iPhone," and you then concluded, "Ok, I guess that means you aren't spending that birthday money at all."

Because that's not the error in the argument, answer choice (D) is incorrect.

Please let me know if that helps. Thanks!
 lawschoolforme
  • Posts: 33
  • Joined: Oct 15, 2013
|
#12113
That super helped - thank you so much!!!
 PositiveThinker
  • Posts: 49
  • Joined: Dec 24, 2016
|
#34899
Answer choice C basically says "it takes the research teams word for it and it is possible that the research team was lying thru their teeth."

Are there any examples of a Flaw method of reasoning question where the correct answer was "the possibility that so and so was lying was overlooked."

Because in my mind in order to draw the conclusions that the donors came to, they'd have to have assumed the researchers were telling the truth. Sounds like reasonable flaw to me. Or is this one that will NEVER be the correct flaw question??

Thanks a lot!
User avatar
 Jonathan Evans
PowerScore Staff
  • PowerScore Staff
  • Posts: 726
  • Joined: Jun 09, 2016
|
#35031
Hi, Positive Thinker,

Good question!

I want to direct you in part to review the general advice I shared in my reply here: lsat/viewtopic.php?f=687&t=1815

With that general analysis/process advice in mind, let's take a closer look at this particular problem. Focus specifically on the conclusion that the foundation made as justification for withdrawing its threat, "that the weapons research would not benefit from the foundation's grant." They base this conclusion on the premise that the university would not use its (the foundation's) funds for weapons research.

The question instructs us to describe a flaw in the foundation's reasoning. Here's the critical issue. Whether or not the department was lying is immaterial to the strength or weakness of the foundation's argument. We are tasked with analyzing simply whether the premise upon which the foundation has based its conclusion offers sufficient support for the foundation's conclusion.

In other words, assuming that the reasons behind the conclusion are accurate (as far as we know), what is the problem with the conclusion.

This problem (flaw) is integral to this argument itself. It is a gap between the premise about redirecting money and the conclusion about the funds not benefiting weapons research.

Certainly, depending on the structure of a different argument, a flaw could involve unreliable testimony or the possibility of dishonesty (survey/sample problems come to mind), but that's not really what's happening here.

Great questions!
User avatar
 emilyjmyer
  • Posts: 48
  • Joined: May 11, 2022
|
#96985
Hi!

Can someone please elaborate on why B is incorrect? Is it because it does not directly relate to the conclusion that the foundation's money will not be used to weapon's research?

For example: say that the university also funded research biohazards (something that is not directly related to a humanitarian aim) does this not affect the conclusion in the argument? And, therefore is not a flaw in the reasoning?

Thanks!
 Adam Tyson
PowerScore Staff
  • PowerScore Staff
  • Posts: 5153
  • Joined: Apr 14, 2011
|
#97265
Your first analysis is on the nose, Emily!
Is it because it does not directly relate to the conclusion that the foundation's money will not be used to weapon's research?
Bingo! The argument did not conclude that the money will only be used for things with a humanitarian purpose, but just that it won't benefit weapons research. It could, however, benefit weapons research by freeing up other funds that could be used that way.

Get the most out of your LSAT Prep Plus subscription.

Analyze and track your performance with our Testing and Analytics Package.